3
$\begingroup$

I am trying to maximize the following function

$$ f(m,n) = \frac{m \log 3 + n \log 2}{\sqrt{m^2+n^2}} $$ where $ n $ and $ m $ are integers, not both $ = 0 $, although one could be $ 0 $.

This is not for a class- it's part of a research problem, and this is a "toy" case. I don't have any background in integer programming, especially nonlinear.

I'm hoping for an proof, that doesn't involve writing a computer program to solve it, if possible.

$\endgroup$
0

1 Answer 1

2
$\begingroup$

Let $a=\log3/\log2$. The problem is equivalent to maximize $$ g(m,n)=\frac{m\,a+n}{\sqrt{m^2+n^2}}. $$From the inequality $$ a\,m+n\le\sqrt{a^2+1}\,\sqrt{m^2+n^2} $$ it follows that $g(m,n)\le\sqrt{a^2+1}$. If there exist an integer $n$ such that $m=n\,a$ is also an integer, then $g(m,n)=\sqrt{a^2+1}$, and this is the maximum value. This works if $a$ is rational. If $a$ is irrational the maximun value is never attained. However it is possible to get as close to it as desired. Let $p_k$, $q_k$ be positive integers, relatively prime, such that $p_k/q_k$ approaches $a$ as $k\to\infty$. Let $r_k=p_k-q_k\,a$. Then $$\begin{align} g(p_k,q_k)&=\frac{p_k\,a+q_k}{\sqrt{p_k^2+q_k^2}}\\ &=\frac{a^2+1}{\sqrt{\Bigl(a+\dfrac{r_k}{q_k}\Bigr)^2+1}}+\frac{r_k}{\sqrt{p_k^2+q_k^2}} \end{align}$$ and $r_k/q_k=p_k/q_k-a$ goes to $0$ as $k\to\infty$.

$\endgroup$

You must log in to answer this question.

Start asking to get answers

Find the answer to your question by asking.

Ask question

Explore related questions

See similar questions with these tags.